1
$\begingroup$

I've written the following proof:

Let $A,B$ be observables (Hermitian operators). define $ ΔA:=A-<A> $. the uncertainty principle is: $$ ⟨(ΔA)^2⟩⟨(ΔB)^2⟩ ≥|⟨[A,B]⟩|^2/4 $$
The Heisenberg equation for $A$ gives: $$ i d/dt ⟨(ΔA)^2 ⟩=⟨[A^2,H]⟩-2⟨A⟩⟨[A,H]⟩ $$ so: $$i d/dt⟨(ΔA)^2⟩⟨(ΔB)^2⟩ = (⟨[A^2,H]⟩-2⟨A⟩⟨[A,H]⟩)⟨(ΔB)^2⟩ + A↔B $$ define $$ C^2:=-i[A^2-2⟨A⟩A,H] , D^2:=-i[B^2-2⟨B⟩B,H] $$ which are Hermitian (are observables and have square roots). using the uncertainty principle for $(C,B)$ and $(D,A)$, we get: $$ d/dt ⟨(ΔA)^2 ⟩⟨(ΔB)^2 ⟩≥|⟨[C,B]⟩|^2/4+|⟨[D,A]⟩|^2/4+⟨C⟩^2⟨(ΔB)^2 ⟩+⟨(ΔA)^2 ⟩⟨D⟩^2 $$
zero is assumed if $A$ or $B$ are constants of motion. so: $$ d/dt ⟨(ΔA)^2 ⟩⟨(ΔB)^2 ⟩≥0 $$ but this is wrong (check out Does Heisenberg's uncertainty under time evolution always grow?). I'm having trouble figuring out what's exactly wrong with my proof? does it only apply when $C,D$ are positive definite?

$\endgroup$
4
  • $\begingroup$ I suspect the issue is that $C^2$ being Hermitian does not imply that $C$ is Hermitian. $\endgroup$ Commented Sep 6 at 20:08
  • $\begingroup$ @J.Delaney is it when C^2 is positive definite? (using the square root of its spectrum in C's spectral decomposition) $\endgroup$ Commented Sep 7 at 5:48
  • 1
    $\begingroup$ Yes. But of course assuming that $C^2$ is positive definite is the same as assuming that $d/dt\langle(\Delta A)^2\rangle >0$, which makes the result a bit trivial. $\endgroup$ Commented Sep 7 at 9:16
  • $\begingroup$ @J.Delaney thanks. $\endgroup$ Commented Sep 7 at 12:38

0

Your Answer

By clicking “Post Your Answer”, you agree to our terms of service and acknowledge you have read our privacy policy.

Start asking to get answers

Find the answer to your question by asking.

Ask question

Explore related questions

See similar questions with these tags.